There is a vector $y in mathbb{R^m}$ such that $A^Ty=0$ and $y^Tb not=0$












1












$begingroup$


Let there be matrix $A_{mtimes n}$ and $b in mathbb{R^m}$ , then either there exists a vector $x in mathbb{R^n}$ such that $Ax=b$ or there is a vector $y in mathbb{R^m}$ such that $A^Ty=0$ and $y^Tb not=0$



I know that if rank of of $A$ is $n$, then we will have the $1$st case :there exists a vector $x in mathbb{R^n}$ such that $Ax=b$; Now for the case rank $A <n$, how to approach? I did some: rank $A <n$ implies there is some $x not=0 in N(A)$ and we have $N(A)$ is orthogonal to $R(A^T)$. Thereafter, I need some help.










share|cite|improve this question











$endgroup$












  • $begingroup$
    if rank of $A$ is $n$, still you can't guarantee $Ax=b$ has solution!! if rank $A$ =m then $Ax=b$ always has solution
    $endgroup$
    – Red shoes
    Jun 3 '17 at 7:52










  • $begingroup$
    Are you familiar with linear programing and duality or Alternative theorems? If yes , I will solve this for you in less than three lines
    $endgroup$
    – Red shoes
    Jun 3 '17 at 8:04










  • $begingroup$
    I know some of LPP. I want to know the proof of the $1$st comment
    $endgroup$
    – Hirakjyoti Das
    Jun 3 '17 at 8:37










  • $begingroup$
    Never mind, I'll write an elementary proof below
    $endgroup$
    – Red shoes
    Jun 3 '17 at 8:49
















1












$begingroup$


Let there be matrix $A_{mtimes n}$ and $b in mathbb{R^m}$ , then either there exists a vector $x in mathbb{R^n}$ such that $Ax=b$ or there is a vector $y in mathbb{R^m}$ such that $A^Ty=0$ and $y^Tb not=0$



I know that if rank of of $A$ is $n$, then we will have the $1$st case :there exists a vector $x in mathbb{R^n}$ such that $Ax=b$; Now for the case rank $A <n$, how to approach? I did some: rank $A <n$ implies there is some $x not=0 in N(A)$ and we have $N(A)$ is orthogonal to $R(A^T)$. Thereafter, I need some help.










share|cite|improve this question











$endgroup$












  • $begingroup$
    if rank of $A$ is $n$, still you can't guarantee $Ax=b$ has solution!! if rank $A$ =m then $Ax=b$ always has solution
    $endgroup$
    – Red shoes
    Jun 3 '17 at 7:52










  • $begingroup$
    Are you familiar with linear programing and duality or Alternative theorems? If yes , I will solve this for you in less than three lines
    $endgroup$
    – Red shoes
    Jun 3 '17 at 8:04










  • $begingroup$
    I know some of LPP. I want to know the proof of the $1$st comment
    $endgroup$
    – Hirakjyoti Das
    Jun 3 '17 at 8:37










  • $begingroup$
    Never mind, I'll write an elementary proof below
    $endgroup$
    – Red shoes
    Jun 3 '17 at 8:49














1












1








1


1



$begingroup$


Let there be matrix $A_{mtimes n}$ and $b in mathbb{R^m}$ , then either there exists a vector $x in mathbb{R^n}$ such that $Ax=b$ or there is a vector $y in mathbb{R^m}$ such that $A^Ty=0$ and $y^Tb not=0$



I know that if rank of of $A$ is $n$, then we will have the $1$st case :there exists a vector $x in mathbb{R^n}$ such that $Ax=b$; Now for the case rank $A <n$, how to approach? I did some: rank $A <n$ implies there is some $x not=0 in N(A)$ and we have $N(A)$ is orthogonal to $R(A^T)$. Thereafter, I need some help.










share|cite|improve this question











$endgroup$




Let there be matrix $A_{mtimes n}$ and $b in mathbb{R^m}$ , then either there exists a vector $x in mathbb{R^n}$ such that $Ax=b$ or there is a vector $y in mathbb{R^m}$ such that $A^Ty=0$ and $y^Tb not=0$



I know that if rank of of $A$ is $n$, then we will have the $1$st case :there exists a vector $x in mathbb{R^n}$ such that $Ax=b$; Now for the case rank $A <n$, how to approach? I did some: rank $A <n$ implies there is some $x not=0 in N(A)$ and we have $N(A)$ is orthogonal to $R(A^T)$. Thereafter, I need some help.







linear-algebra matrices






share|cite|improve this question















share|cite|improve this question













share|cite|improve this question




share|cite|improve this question








edited Dec 10 '18 at 10:50









José Carlos Santos

157k22126227




157k22126227










asked Jun 3 '17 at 7:22









Hirakjyoti DasHirakjyoti Das

389112




389112












  • $begingroup$
    if rank of $A$ is $n$, still you can't guarantee $Ax=b$ has solution!! if rank $A$ =m then $Ax=b$ always has solution
    $endgroup$
    – Red shoes
    Jun 3 '17 at 7:52










  • $begingroup$
    Are you familiar with linear programing and duality or Alternative theorems? If yes , I will solve this for you in less than three lines
    $endgroup$
    – Red shoes
    Jun 3 '17 at 8:04










  • $begingroup$
    I know some of LPP. I want to know the proof of the $1$st comment
    $endgroup$
    – Hirakjyoti Das
    Jun 3 '17 at 8:37










  • $begingroup$
    Never mind, I'll write an elementary proof below
    $endgroup$
    – Red shoes
    Jun 3 '17 at 8:49


















  • $begingroup$
    if rank of $A$ is $n$, still you can't guarantee $Ax=b$ has solution!! if rank $A$ =m then $Ax=b$ always has solution
    $endgroup$
    – Red shoes
    Jun 3 '17 at 7:52










  • $begingroup$
    Are you familiar with linear programing and duality or Alternative theorems? If yes , I will solve this for you in less than three lines
    $endgroup$
    – Red shoes
    Jun 3 '17 at 8:04










  • $begingroup$
    I know some of LPP. I want to know the proof of the $1$st comment
    $endgroup$
    – Hirakjyoti Das
    Jun 3 '17 at 8:37










  • $begingroup$
    Never mind, I'll write an elementary proof below
    $endgroup$
    – Red shoes
    Jun 3 '17 at 8:49
















$begingroup$
if rank of $A$ is $n$, still you can't guarantee $Ax=b$ has solution!! if rank $A$ =m then $Ax=b$ always has solution
$endgroup$
– Red shoes
Jun 3 '17 at 7:52




$begingroup$
if rank of $A$ is $n$, still you can't guarantee $Ax=b$ has solution!! if rank $A$ =m then $Ax=b$ always has solution
$endgroup$
– Red shoes
Jun 3 '17 at 7:52












$begingroup$
Are you familiar with linear programing and duality or Alternative theorems? If yes , I will solve this for you in less than three lines
$endgroup$
– Red shoes
Jun 3 '17 at 8:04




$begingroup$
Are you familiar with linear programing and duality or Alternative theorems? If yes , I will solve this for you in less than three lines
$endgroup$
– Red shoes
Jun 3 '17 at 8:04












$begingroup$
I know some of LPP. I want to know the proof of the $1$st comment
$endgroup$
– Hirakjyoti Das
Jun 3 '17 at 8:37




$begingroup$
I know some of LPP. I want to know the proof of the $1$st comment
$endgroup$
– Hirakjyoti Das
Jun 3 '17 at 8:37












$begingroup$
Never mind, I'll write an elementary proof below
$endgroup$
– Red shoes
Jun 3 '17 at 8:49




$begingroup$
Never mind, I'll write an elementary proof below
$endgroup$
– Red shoes
Jun 3 '17 at 8:49










2 Answers
2






active

oldest

votes


















2












$begingroup$

Let $ Ax=b$ has no solution, thus $b notin R(A).$ we know that $R^m = R(A)^{perp} + R(A)$. So $$b=y+Ax$$ for some nonzero $y in R(A)^{perp}=N(A^T) $ and some $xin R^n$. Hence ; $$A^Ty=0$$ and $$y^Tb=|y|^2+y^TAx = |y|^2 neq 0$$
.






share|cite|improve this answer











$endgroup$













  • $begingroup$
    that's very helpful, thank you
    $endgroup$
    – Hirakjyoti Das
    Jun 3 '17 at 8:55





















0












$begingroup$

Imagine that you try to solve the equation $Ax=b$. You can do it by Gaussian elimination. In the end, you get an equation of the type $A'x=b'$, whre $A'$ is something like$$begin{pmatrix}alpha_1&*&*&*&*&*\0&0&alpha_2&*&*&*\0&0&0&alpha_3&*&*\0&0&0&0&0&0\0&0&0&0&0&0end{pmatrix},$$with every $alpha_kneq0$. So, the system has a solution if and only if the last two coordinates of $b'$ are equal to $0$. Otherwise, take $y=(0,0,0,beta_4,beta_5)$, where $beta_4$ and $beta_5$ are the two last coordinates of $b'$.






share|cite|improve this answer









$endgroup$













    Your Answer





    StackExchange.ifUsing("editor", function () {
    return StackExchange.using("mathjaxEditing", function () {
    StackExchange.MarkdownEditor.creationCallbacks.add(function (editor, postfix) {
    StackExchange.mathjaxEditing.prepareWmdForMathJax(editor, postfix, [["$", "$"], ["\\(","\\)"]]);
    });
    });
    }, "mathjax-editing");

    StackExchange.ready(function() {
    var channelOptions = {
    tags: "".split(" "),
    id: "69"
    };
    initTagRenderer("".split(" "), "".split(" "), channelOptions);

    StackExchange.using("externalEditor", function() {
    // Have to fire editor after snippets, if snippets enabled
    if (StackExchange.settings.snippets.snippetsEnabled) {
    StackExchange.using("snippets", function() {
    createEditor();
    });
    }
    else {
    createEditor();
    }
    });

    function createEditor() {
    StackExchange.prepareEditor({
    heartbeatType: 'answer',
    autoActivateHeartbeat: false,
    convertImagesToLinks: true,
    noModals: true,
    showLowRepImageUploadWarning: true,
    reputationToPostImages: 10,
    bindNavPrevention: true,
    postfix: "",
    imageUploader: {
    brandingHtml: "Powered by u003ca class="icon-imgur-white" href="https://imgur.com/"u003eu003c/au003e",
    contentPolicyHtml: "User contributions licensed under u003ca href="https://creativecommons.org/licenses/by-sa/3.0/"u003ecc by-sa 3.0 with attribution requiredu003c/au003e u003ca href="https://stackoverflow.com/legal/content-policy"u003e(content policy)u003c/au003e",
    allowUrls: true
    },
    noCode: true, onDemand: true,
    discardSelector: ".discard-answer"
    ,immediatelyShowMarkdownHelp:true
    });


    }
    });














    draft saved

    draft discarded


















    StackExchange.ready(
    function () {
    StackExchange.openid.initPostLogin('.new-post-login', 'https%3a%2f%2fmath.stackexchange.com%2fquestions%2f2307863%2fthere-is-a-vector-y-in-mathbbrm-such-that-aty-0-and-ytb-not-0%23new-answer', 'question_page');
    }
    );

    Post as a guest















    Required, but never shown

























    2 Answers
    2






    active

    oldest

    votes








    2 Answers
    2






    active

    oldest

    votes









    active

    oldest

    votes






    active

    oldest

    votes









    2












    $begingroup$

    Let $ Ax=b$ has no solution, thus $b notin R(A).$ we know that $R^m = R(A)^{perp} + R(A)$. So $$b=y+Ax$$ for some nonzero $y in R(A)^{perp}=N(A^T) $ and some $xin R^n$. Hence ; $$A^Ty=0$$ and $$y^Tb=|y|^2+y^TAx = |y|^2 neq 0$$
    .






    share|cite|improve this answer











    $endgroup$













    • $begingroup$
      that's very helpful, thank you
      $endgroup$
      – Hirakjyoti Das
      Jun 3 '17 at 8:55


















    2












    $begingroup$

    Let $ Ax=b$ has no solution, thus $b notin R(A).$ we know that $R^m = R(A)^{perp} + R(A)$. So $$b=y+Ax$$ for some nonzero $y in R(A)^{perp}=N(A^T) $ and some $xin R^n$. Hence ; $$A^Ty=0$$ and $$y^Tb=|y|^2+y^TAx = |y|^2 neq 0$$
    .






    share|cite|improve this answer











    $endgroup$













    • $begingroup$
      that's very helpful, thank you
      $endgroup$
      – Hirakjyoti Das
      Jun 3 '17 at 8:55
















    2












    2








    2





    $begingroup$

    Let $ Ax=b$ has no solution, thus $b notin R(A).$ we know that $R^m = R(A)^{perp} + R(A)$. So $$b=y+Ax$$ for some nonzero $y in R(A)^{perp}=N(A^T) $ and some $xin R^n$. Hence ; $$A^Ty=0$$ and $$y^Tb=|y|^2+y^TAx = |y|^2 neq 0$$
    .






    share|cite|improve this answer











    $endgroup$



    Let $ Ax=b$ has no solution, thus $b notin R(A).$ we know that $R^m = R(A)^{perp} + R(A)$. So $$b=y+Ax$$ for some nonzero $y in R(A)^{perp}=N(A^T) $ and some $xin R^n$. Hence ; $$A^Ty=0$$ and $$y^Tb=|y|^2+y^TAx = |y|^2 neq 0$$
    .







    share|cite|improve this answer














    share|cite|improve this answer



    share|cite|improve this answer








    edited Jun 3 '17 at 8:53

























    answered Jun 3 '17 at 8:45









    Red shoesRed shoes

    4,744621




    4,744621












    • $begingroup$
      that's very helpful, thank you
      $endgroup$
      – Hirakjyoti Das
      Jun 3 '17 at 8:55




















    • $begingroup$
      that's very helpful, thank you
      $endgroup$
      – Hirakjyoti Das
      Jun 3 '17 at 8:55


















    $begingroup$
    that's very helpful, thank you
    $endgroup$
    – Hirakjyoti Das
    Jun 3 '17 at 8:55






    $begingroup$
    that's very helpful, thank you
    $endgroup$
    – Hirakjyoti Das
    Jun 3 '17 at 8:55













    0












    $begingroup$

    Imagine that you try to solve the equation $Ax=b$. You can do it by Gaussian elimination. In the end, you get an equation of the type $A'x=b'$, whre $A'$ is something like$$begin{pmatrix}alpha_1&*&*&*&*&*\0&0&alpha_2&*&*&*\0&0&0&alpha_3&*&*\0&0&0&0&0&0\0&0&0&0&0&0end{pmatrix},$$with every $alpha_kneq0$. So, the system has a solution if and only if the last two coordinates of $b'$ are equal to $0$. Otherwise, take $y=(0,0,0,beta_4,beta_5)$, where $beta_4$ and $beta_5$ are the two last coordinates of $b'$.






    share|cite|improve this answer









    $endgroup$


















      0












      $begingroup$

      Imagine that you try to solve the equation $Ax=b$. You can do it by Gaussian elimination. In the end, you get an equation of the type $A'x=b'$, whre $A'$ is something like$$begin{pmatrix}alpha_1&*&*&*&*&*\0&0&alpha_2&*&*&*\0&0&0&alpha_3&*&*\0&0&0&0&0&0\0&0&0&0&0&0end{pmatrix},$$with every $alpha_kneq0$. So, the system has a solution if and only if the last two coordinates of $b'$ are equal to $0$. Otherwise, take $y=(0,0,0,beta_4,beta_5)$, where $beta_4$ and $beta_5$ are the two last coordinates of $b'$.






      share|cite|improve this answer









      $endgroup$
















        0












        0








        0





        $begingroup$

        Imagine that you try to solve the equation $Ax=b$. You can do it by Gaussian elimination. In the end, you get an equation of the type $A'x=b'$, whre $A'$ is something like$$begin{pmatrix}alpha_1&*&*&*&*&*\0&0&alpha_2&*&*&*\0&0&0&alpha_3&*&*\0&0&0&0&0&0\0&0&0&0&0&0end{pmatrix},$$with every $alpha_kneq0$. So, the system has a solution if and only if the last two coordinates of $b'$ are equal to $0$. Otherwise, take $y=(0,0,0,beta_4,beta_5)$, where $beta_4$ and $beta_5$ are the two last coordinates of $b'$.






        share|cite|improve this answer









        $endgroup$



        Imagine that you try to solve the equation $Ax=b$. You can do it by Gaussian elimination. In the end, you get an equation of the type $A'x=b'$, whre $A'$ is something like$$begin{pmatrix}alpha_1&*&*&*&*&*\0&0&alpha_2&*&*&*\0&0&0&alpha_3&*&*\0&0&0&0&0&0\0&0&0&0&0&0end{pmatrix},$$with every $alpha_kneq0$. So, the system has a solution if and only if the last two coordinates of $b'$ are equal to $0$. Otherwise, take $y=(0,0,0,beta_4,beta_5)$, where $beta_4$ and $beta_5$ are the two last coordinates of $b'$.







        share|cite|improve this answer












        share|cite|improve this answer



        share|cite|improve this answer










        answered Jun 3 '17 at 8:07









        José Carlos SantosJosé Carlos Santos

        157k22126227




        157k22126227






























            draft saved

            draft discarded




















































            Thanks for contributing an answer to Mathematics Stack Exchange!


            • Please be sure to answer the question. Provide details and share your research!

            But avoid



            • Asking for help, clarification, or responding to other answers.

            • Making statements based on opinion; back them up with references or personal experience.


            Use MathJax to format equations. MathJax reference.


            To learn more, see our tips on writing great answers.




            draft saved


            draft discarded














            StackExchange.ready(
            function () {
            StackExchange.openid.initPostLogin('.new-post-login', 'https%3a%2f%2fmath.stackexchange.com%2fquestions%2f2307863%2fthere-is-a-vector-y-in-mathbbrm-such-that-aty-0-and-ytb-not-0%23new-answer', 'question_page');
            }
            );

            Post as a guest















            Required, but never shown





















































            Required, but never shown














            Required, but never shown












            Required, but never shown







            Required, but never shown

































            Required, but never shown














            Required, but never shown












            Required, but never shown







            Required, but never shown







            Popular posts from this blog

            To store a contact into the json file from server.js file using a class in NodeJS

            Redirect URL with Chrome Remote Debugging Android Devices

            Dieringhausen